Jump to content

Frankesten's Content

There have been 59 items by Frankesten (Search limited from 08-06-2020)



Sort by                Order  

#576498 $\sum \frac{a^{2}+bc}{b+ca}...

Posted by Frankesten on 29-07-2015 - 15:33 in Bất đẳng thức và cực trị

Với a,b,c dương và $\sum  a =3$. CMR: $\sum \frac{a^{2}+bc}{b+ca} \geq 3$




#576204 $\sum (\frac{a^{3}}{a^{3}+(...

Posted by Frankesten on 28-07-2015 - 16:10 in Bất đẳng thức và cực trị

Cho a,b,c dương. CMR $\sum (\frac{a^{3}}{a^{3}+(b+c)^{3}})^{\frac{1}{2}} \geq 1$




#576202 $\sum a^{2}b* \sum ab \leq 9$

Posted by Frankesten on 28-07-2015 - 16:05 in Bất đẳng thức và cực trị

Cho a,b,c>=0; $\sum a =3$ ; CMR: $(a^{2}b+b^{2}c+c^{2}a)(ab+bc+ca) \leq 9$




#576196 $\sum {a^{2}+b}+abc \leq 4$

Posted by Frankesten on 28-07-2015 - 15:58 in Bất đẳng thức và cực trị

Cho a,b,c dương, a+b+c=3. CMR: $a^{2}b+b^{2}c+c^{2}a+abc \leq 4$




#576190 $\sum \frac{a^{2}+bc}{b+ca}...

Posted by Frankesten on 28-07-2015 - 15:51 in Bất đẳng thức và cực trị

 

$\Leftrightarrow a+bc-b^{2}+abc\geq 0$(Cái này quy đồng rồi đặt $a^{2}$ làm nhân tử chung nha)

Tới đây với nhận xét $a,b,c\leq 1$ ta suy ra nó đúng

Suy ra đpcm

tại sao a,b,c<= 1 vậy? bạn có thể giải thích rõ hơn ko?




#576187 $\sum ab + \frac{1}{abc} \geq abc+3...

Posted by Frankesten on 28-07-2015 - 15:46 in Bất đẳng thức và cực trị

Cho a+b+c=3, a,b,c >0. CMR: $\sum ab + \frac{1}{abc} \geq abc+3$




#576165 $\sum \frac{a}{b} \geq 3 + \frac...

Posted by Frankesten on 28-07-2015 - 15:01 in Bất đẳng thức và cực trị

cho a,b,c dương. CMR: $\sum \frac{a}{b} \geq 3 + \frac{(c-a)^{2}}{ab+bc+ca}$




#576164 $\sum \frac{a}{b^{3}+16} \g...

Posted by Frankesten on 28-07-2015 - 14:54 in Bất đẳng thức và cực trị

Cho a+b+c=3. CMR: $\sum \frac{a}{b^{3}+16} \geq \frac{1}{6}$ . đẳng thức xảy ra tại a=0, b=1, c=2, vv




#576161 $\sum \frac{a^{2}+bc}{b+ca}...

Posted by Frankesten on 28-07-2015 - 14:42 in Bất đẳng thức và cực trị

Cho a+b+c=3.a,b,c dương CMR:

 

$\frac{a^{2}+bc}{b+ca}+\frac{b^{2}+ca}{c+ab}+\frac{c^{2}+ab}{a+bc}\geq3$